se eu por k=oo o pn sempre vai ser menor do que n^oo, tem que provar que e
valido para k=2?

On Mon, Apr 7, 2008 at 10:50 AM, Artur Costa Steiner <
[EMAIL PROTECTED]> wrote:

>   Acho este problema interessante. Encontrei uma solucao, gostaria de ver
> como os colegas resolvem.
>
> Seja p_n, n =1,2,3.... a sequencia dos numeros primos. Mostre que, para
> todo k > 1, a desigualdade,
>
> p_n < n^k
>
> ocorre para uma infinidade de índices n.
>

Responder a